Integrating Problem: Definite Integral from sqrt2 to 2

  • Thread starter Thread starter miller8605
  • Start date Start date
  • Tags Tags
    Intergration
Click For Summary
The discussion centers on evaluating the definite integral of the function 1/(x^3√(x^2-1)) from √2 to 2. The user has attempted to separate the integrand into two parts, using trigonometric substitution for the second part, and is uncertain about the integration of 1/x^3. There is a suggestion that the entire integrand can be simplified more effectively with a single trigonometric substitution rather than separating it. Clarification is requested regarding the user's method, indicating some confusion about the integration process. The conversation highlights the importance of a cohesive approach to solving the integral.
miller8605
Messages
17
Reaction score
0

Homework Statement


i'm taking a defitinite integral from sqrt2 to 2 of the function 1/x^3*sqrt(x^2-1)dx.

Homework Equations





The Attempt at a Solution


I separated it into 1/x^3 and 1/sqrt(x^2-1). I have the second part using trig sub. as being sec theta dtheta, before integrating it. I believe i did this part correctly.

What I can't remember is that I make 1/x^3 to x^-3 and then integrate it that way with the final being -1/2(1/x^2)??

Am i going about this correctly?
 
Physics news on Phys.org
miller8605 said:
I separated it into 1/x^3 and 1/sqrt(x^2-1).

No need to do that. The entire integrand comes out very nice with a trig substitution.

I have the second part using trig sub. as being sec theta dtheta[/color], before integrating it. I believe i did this part correctly.

The part in red[/color] makes very little sense to me, but it sounds like it has the kernel of a correct method in there. Could you elaborate?
 
Question: A clock's minute hand has length 4 and its hour hand has length 3. What is the distance between the tips at the moment when it is increasing most rapidly?(Putnam Exam Question) Answer: Making assumption that both the hands moves at constant angular velocities, the answer is ## \sqrt{7} .## But don't you think this assumption is somewhat doubtful and wrong?

Similar threads

  • · Replies 105 ·
4
Replies
105
Views
6K
  • · Replies 2 ·
Replies
2
Views
1K
Replies
4
Views
2K
  • · Replies 10 ·
Replies
10
Views
2K
  • · Replies 22 ·
Replies
22
Views
3K
  • · Replies 1 ·
Replies
1
Views
2K
  • · Replies 12 ·
Replies
12
Views
2K
  • · Replies 5 ·
Replies
5
Views
2K
  • · Replies 6 ·
Replies
6
Views
2K
  • · Replies 14 ·
Replies
14
Views
2K